[Date Prev][Date Next][Thread Prev][Thread Next][Date Index][Thread Index]

[obm-l] teorema de fermat



Um colega outro dia me disse que não seria tão difícil demostrar o último teorema de fermat para o caso n = 4, a saber:

        Não existe uma tripla de inteiros (x, y, z), para n > 2, que satisfaça a equação:

              x^n + y^n = z^n.

No entanto não consegui resolver tal problema... Se alguém puder me ajudar, agradeço!



Busca Yahoo!
O serviço de busca mais completo da Internet. O que você pensar o Yahoo! encontra.